Thursday 17 March 2016

real analysis - Show that $f$ is continuous at exactly one point




Let $f:\mathbb{R}\to\mathbb{R}$ be defined by



$$f(x)=
\begin{cases}
5x+7 & \text{ if } x \text{ is rational } \\
x+11 & \text{ if } x \text{ is irrational }

\end{cases}$$



Show that f is continuous at exactly one point.




My attempt:
Let $a \in \mathbb R$, and take a rational sequence $(x_n)$ and an irrational sequence $(y_n)$ such that $x_n \to a$ and $y_n \to a$.
Then $f(x_n) = 5x_n + 7 \to 5a+7$ and $f(y_n) = y_n + 11 \to a + 11$.
We have $lim_{n \to \infty}f(x_n) = lim_{n \to \infty}f(y_n) = f(a) \iff a = 1$.
Therefore $f$ is continuous only at $x = 1$.



Is the above valid? In the above, I have shown that for every sequence $(z_n)$ such that $(z_n) \to 1$, $lim_{n \to \infty} f(z_n) = f(1)$. Doesn't that prove that $f$ is continuous at $x = 1$? In the given solutions they also used the $\epsilon - \delta$ definition of continuity to prove that $f$ is continuous at $x = 1$, I'm not sure if that is necessary, or why it is necessary. Can anyone explain?




Thanks.


Answer



Using the sequential criterion for continuity is fine, but I think you have to make clear, why $\bigl(f(z_n)\bigr)$ converges for all $z_n \to 1$, even when not all $z_n$ are rational or all irrational. That almost follows from your argument (just look at the subseqeunces of the rational resp. irrational $z_n$), but you can expand on this a little:




Let $z_n \to 1$ be any sequence. If all but finitely many $z_n$ are rational, then $f(z_n) \to f(1)$, as allready shown (the finitely many terms do not make a difference), same if all but finitely many $z_n$ are irrational. Otherwise (that is we have infinitely many rational and irrational $z_n$) let $(z_{n_k})$ denote the subsequence of rational and $(z_{n'_k})$ the subsequence of irrational numbers. Then $(z_{n_k})$ and $(z_{n'_k})$ cover $(z_n)$ and have the same limit $f(1)$ (as shown), hence $f(z_n) \to f(1)$.



No comments:

Post a Comment

real analysis - How to find $lim_{hrightarrow 0}frac{sin(ha)}{h}$

How to find $\lim_{h\rightarrow 0}\frac{\sin(ha)}{h}$ without lhopital rule? I know when I use lhopital I easy get $$ \lim_{h\rightarrow 0}...